Eliminate constants/Moment of Inertia

  • Thread starter aaronfue
  • Start date
  • Tags
    Inertia
In summary, the problem involves finding the moment of inertia, Iy, about the y-axis using a vertical strip of width dx. The given equation is ∫A x2 dA, but the constants k and c need to be eliminated. The solution involves finding two points on the curve to obtain two equations in two unknowns (c and k). The final answer should be in terms of a number and powers of constants a and b.
  • #1
aaronfue
122
0

Homework Statement



Find the moment of inertia, Iy, about the y-axis using a vertical strip, width dx.
Use the given equation and eliminate constants k and c, express answer in terms of number and powers of a and b.


Homework Equations



I can't figure out how to eliminate the constants k and c?

The Attempt at a Solution



I believe I have the integration ready:

[itex]\int^{a}_{0}[/itex] x[itex]^{2}[/itex] (b-y)dx
where y=k(x-a)2 + c
 

Attachments

  • Moment of Inertia.JPG
    Moment of Inertia.JPG
    17.6 KB · Views: 537
Physics news on Phys.org
  • #2
aaronfue said:
I can't figure out how to eliminate the constants k and c?

Pick out two points on the curve where you know the coordinates. Use these values to obtain two equations in two unknowns (c and k).
 
  • #3
Ok. So far this is what I have:

I know the point at the origin (0,0). And x=a and y=b.

0 = k(0 - a)2 + c = ka2 + c ==> k = -[itex]\frac{c}{a^2}[/itex]

So,

y=(-[itex]\frac{c}{a^2}[/itex])(x-a)2 + c

I'm still not sure how to get rid of the c?
 
  • #4
Start with your second known point.
 
  • #5
I see... so x=a, y=b

b=(-[itex]\frac{c}{a^2}[/itex])(a-a)2 + c
b=(-[itex]\frac{c}{a^2}[/itex])(0)2 + c
b=c

...I think??
 
  • #6
aaronfue said:
I see... so x=a, y=b

b=(-[itex]\frac{c}{a^2}[/itex])(a-a)2 + c
b=(-[itex]\frac{c}{a^2}[/itex])(0)2 + c
b=c

...I think??

Yup. So replace c in the y(x) function and then plug in the (0,0) point to see what k is all about...
 
  • #7
gneill said:
Yup. So replace c in the y(x) function and then plug in the (0,0) point to see what k is all about...

Thanks! Would you also check if my integration is set up correctly?

[itex]\int^{a}_{0}[/itex] x2 [b-([itex]\frac{-b}{a^2}[/itex])(x-a2) + b] dx

How would I get my answer in terms of a number and powers of constants a and b (per instructions)?
 
Last edited:
  • #8
aaronfue said:
Thanks! Would you also check if my integration is set up correctly?

[itex]\int^{a}_{0}[/itex] x2 [b-([itex]\frac{-b}{a^2}[/itex])(x-a2) + b] dx

How would I get my answer in terms of a number and powers of constants a and b (per instructions)?

For your integration, be sure that you're squaring the right thing in the integrand; is it just "a" that is squared?

Once you perform the integration the result will be in terms of a constant and powers of a and b.

Note that the tacit assumption is that the 'material' that the figure is made of has a uniform density of 1 mass unit per unit area. If instead you want to assume a mass M for the object, you should calculate its density from M and the total area and incorporate this into the determination of the moment of inertia.
 
  • #9
gneill said:
For your integration, be sure that you're squaring the right thing in the integrand; is it just "a" that is squared?

Once you perform the integration the result will be in terms of a constant and powers of a and b.

Note that the tacit assumption is that the 'material' that the figure is made of has a uniform density of 1 mass unit per unit area. If instead you want to assume a mass M for the object, you should calculate its density from M and the total area and incorporate this into the determination of the moment of inertia.

I thought that the equation for Iy was

A x2 dA

Why would the "a" be the only thing squared?
 
  • #10
Isn't the function ##-\frac{b}{a^2}(x - a)^2 + b##, so that it's the whole term ##(x - a)## that's squared, not just the a in the term?
 
  • #11
gneill said:
Isn't the function ##-\frac{b}{a^2}(x - a)^2 + b##, so that it's the whole term ##(x - a)## that's squared, not just the a in the term?

My answer came out to be:

[itex]\frac{3ba^3}{10}[/itex]

I don't understand how to get a power of a constant with this problem?
 
  • #12
a and b are constants. So in your answer b is to the power of 1 and a is to the power of 3.

The numerical constant 3/10 looks off to me though. Can you show more detail in your work?
 
  • #13
gneill said:
a and b are constants. So in your answer b is to the power of 1 and a is to the power of 3.

The numerical constant 3/10 looks off to me though. Can you show more detail in your work?

I've attached a scan of my work. Now that I've looked at the problem, I think I left out a "b-" for my "ydx". I think that it should be (b-y)dx. Seem right?
 

Attachments

  • MOI #1.JPG
    MOI #1.JPG
    14 KB · Views: 456
Last edited:
  • #14
aaronfue said:
I've attached a scan of my work. Now that I've looked at the problem, I think I left out a "b-" for my "ydx". I think that it should be (b-y)dx. Seem right?

Yup. b-y(x) is the length of the differential strips.
 
  • #15
gneill said:
Yup. b-y(x) is the length of the differential strips.

After including "b-" in my problem my answer came out to be:

[itex]\frac{8ba^3}{15}[/itex] - [itex]\frac{2ba}{3}[/itex],

without factoring.

Radius of Gyration: Ky = [itex]\sqrt{\frac{(\frac{8ba^3}{15} - \frac{2ba}{3})}{(b-y)dx}}[/itex]
 

Attachments

  • Calc's.JPG
    Calc's.JPG
    41.3 KB · Views: 459
  • #16
Your integral should look something like:

$$\int_0^a x^2\left[ b - \left[-\frac{b}{a^2}*(x - a)^2 +b\right]\right]dx$$
$$= \int_0^a x^2\left[\frac{b}{a^2}(x - a)^2\right]dx$$
$$= \int_0^a b x^2 - 2\frac{b}{a}x^3 + \frac{b}{a^2}x^4 \, dx$$

Evaluating the last form term by term should leave you with terms having b to the power of one, a to the power of three only. Try the integral again.
 
  • #17
gneill said:
Your integral should look something like:

$$\int_0^a x^2\left[ b - \left[-\frac{b}{a^2}*(x - a)^2 +b\right]\right]dx$$
$$= \int_0^a x^2\left[\frac{b}{a^2}(x - a)^2\right]dx$$
$$= \int_0^a b x^2 - 2\frac{b}{a}x^3 + \frac{b}{a^2}x^4 \, dx$$

Evaluating the last form term by term should leave you with terms having b to the power of one, a to the power of three only. Try the integral again.

I see what I did wrong. Instead of having (-[itex]\frac{2b}{a}[/itex]x3), I had: (-[itex]\frac{2b}{a^2}[/itex]x3).

I now have: ba2*([itex]\frac{8ba}{15}[/itex] - [itex]\frac{2}{3}[/itex])??

I believe I integrated correctly.
 
Last edited:
  • #18
aaronfue said:
I see what I did wrong. Instead of having (-[itex]\frac{2b}{a}[/itex]x3), I had: (-[itex]\frac{2b}{a^2}[/itex]x3).

I now have: ba2*([itex]\frac{8ba}{15}[/itex] - [itex]\frac{2}{3}[/itex])??

I believe I integrated correctly.

If that's your result for the integration, it's not correct. Integrate term by term and show your work.
 
  • #19
Here is the new integration that I did. I didn't know if I should leave it as is or try to simplify it. There are no instructions that say to do so.
 

Attachments

  • Integration.JPG
    Integration.JPG
    21.6 KB · Views: 444
  • #20
In the first line of your attachment, the term with the coefficient -2b/a should be an x3 term, not x2. There's one x2 term, one x3 term, and one x4 term going into the integration.
 
  • #21
gneill said:
In the first line of your attachment, the term with the coefficient -2b/a should be an x3 term, not x2. There's one x2 term, one x3 term, and one x4 term going into the integration.

Dang...I see! After I distributed the x2 I didn't apply it to that part.
 
  • #22
Alright...

After re-distributing the x2 into my (b-y)dx, I got the answer that is in the image attached.

I guess I could factor out the ba3 and add/subtract fractions...
 

Attachments

  • New eqs..JPG
    New eqs..JPG
    25.6 KB · Views: 424
  • #23
Yup. Simplify the result.
 
  • #24
Great.

Now all I have to do is figure out the same answer using a horizontal strip (dy)!? I'm guessing, solve the equation as "x=" and plug that in?

Use: [itex]\frac{x^3}{3}[/itex]dy
 
  • #25
aaronfue said:
Great.

Now all I have to do is figure out the same answer using a horizontal strip (dy)!? I'm guessing, solve the equation as "x=" and plug that in?

Use: [itex]\frac{x^3}{3}[/itex]dy

Ouch. That might be a bit nasty; your x(y) function is going to involve a radical.

Well, show us what you get :smile:
 
  • #26
gneill said:
Ouch. That might be a bit nasty; your x(y) function is going to involve a radical.

Well, show us what you get :smile:

I was given a suggestion to substitute the expression for dy in terms of x and dx then integrate over the limits of x (0 to a).

Also to use [itex]\frac{x^3}{3}[/itex]dy

Here is what I did:

I took the derivative of my function to get my "dy":

dy = -[itex]\frac{2bx}{a^2}[/itex] + [itex]\frac{2b}{a}[/itex] dx

Then I integrated that to get my final answer of [itex]\frac{ba^3}{30}[/itex], which was the same as I initially got.
 
  • #27
Hey, there's no arguing with success :smile:
 

Related to Eliminate constants/Moment of Inertia

1. What are constants and how do they affect moments of inertia?

Constants are numerical values that remain the same throughout an experiment or calculation. In terms of moments of inertia, constants can affect the value by either increasing or decreasing it based on their specific properties. For example, a constant with a larger numerical value will increase the moment of inertia, while a constant with a smaller numerical value will decrease the moment of inertia.

2. Why is it important to eliminate constants when calculating moments of inertia?

Eliminating constants is important in order to isolate and accurately measure the moment of inertia. If constants are not eliminated, they can skew the results and make it difficult to determine the true moment of inertia value.

3. How do you eliminate constants when calculating moments of inertia?

To eliminate constants, you can either rearrange the equation to eliminate them algebraically or use conversion factors to cancel them out. It is important to carefully follow the rules of algebra and pay attention to units when eliminating constants.

4. Can you provide an example of eliminating constants in a moment of inertia calculation?

Sure, let's say we have an object with a mass of 4 kg and a radius of 2 m rotating about an axis. The moment of inertia equation for a solid sphere is I = (2/5) * m * r^2. If we substitute in the values, we get I = (2/5) * 4 kg * (2 m)^2 = (2/5) * 16 kg*m^2 = 6.4 kg*m^2. In this case, the constant (2/5) is eliminated when we substitute in the values and we are left with the correct moment of inertia for a solid sphere.

5. Are there any situations where it is not necessary to eliminate constants in moments of inertia calculations?

In some cases, constants may not need to be eliminated if they do not significantly affect the value of the moment of inertia. This is usually the case when the constants have a small numerical value or when the values are already known and accounted for in the calculation. However, it is still important to be aware of the constants and their potential impact on the moment of inertia value.

Similar threads

  • Engineering and Comp Sci Homework Help
Replies
27
Views
2K
  • Engineering and Comp Sci Homework Help
Replies
2
Views
2K
  • Engineering and Comp Sci Homework Help
Replies
5
Views
2K
  • Engineering and Comp Sci Homework Help
Replies
7
Views
921
Replies
25
Views
598
  • Mechanics
Replies
2
Views
926
  • Engineering and Comp Sci Homework Help
Replies
15
Views
3K
  • Engineering and Comp Sci Homework Help
Replies
10
Views
2K
  • Engineering and Comp Sci Homework Help
Replies
6
Views
2K
  • Engineering and Comp Sci Homework Help
Replies
3
Views
1K
Back
Top